3x - 7y = 13; x = -3, 0, 3

Solve the equation for y.
and solve for x. ​

Answers

Answer 1

Answer:

when x = -3; y = -22/7

when x = 0;  y = -13/7

when x = 3;  y = -4/7

y = 3/7x - 13/7

x = 7/3y + 13/3

Step-by-step explanation:

x = -3;    3(-3) - 7y = 13; -9 - 7y = 13; -7y = 22; y = -22/7

x = 0;     3(0) -7y = 13; -7y = 13; y = -13/7

x = 3;     3(3) -7y = 13; 9 - 7y = 13; -7y = 4; y = -4/7

SOLVE FOR Y: 3x - 7y = 13; -7y = -3x + 13; y = 3/7x - 13/7

SOLVE FOR X: 3x - 7y = 13; 3x = 7y + 13; x = 7/3y + 13/3


Related Questions

what is the distance between the bank and Hiroto's house on the mapEnter the distance to the nearest quarter mile. (Each unit represents 1 mile. Its due today can anyone help?

Answers

Answer:

Is there a picture to go with the question??

Step-by-step explanation:

A student has a dog and cat for a pet. The dog eats 3 pounds of food
for every 1 pound of food the cat eats. If the pets eat a total of 464
pounds of food combined, how much food did each the cat and dog
eat?

Answers

Answer:

I'm not reallu sure.. but 5

Step-by-step explanation:

Want free points?! Want free brainliest?! Come to my profile and just answer one of my questions. I am YOUR community builder who gives free things. ("Thanks" on ur profile, points, hacks, brainliest, etc.)

Put me as brainliest so I can receive 50% more points!

Write an equation for the line in point slope form that passes through (-3,9) and (11,-2) FAST PLEASE

Answers

The equation is Y= 11/14x + 6.666

what is the x-coordinate of (-2,5)?

Answers

Answer: -2

Explanation: The -2 is called the x-coordinate

or horizontal move from the origin.

All points are written in the form (x, y).

-2


When having an ordered pair the first is the x-coordinate the second is the Y coordinate

(2 x minus 1)(x + 4)
whats the product

Answers

Answer:

2x^2+7x-4

Step-by-step explanation:

2x-x=x the 4+1=5 so x=5

Step-by-step explanation:

James owes His friend Sam $65. He pays Him back $38. How much does James still owe

Answers

Answer:

James still owes the 27 bucks

Answer:

27$

Step-by-step explanation:

65-38

What is the y intercept form of y=5x-1

Choose 1 answer:


(Choice A) (-1,0)

(Choice B) (0,-1)

(Choice C) (5,0)

(Choice D) (0,5)
WILL MARK BRAINLIEST

Answers

Answer:

b

Step-by-step explanation:

y=mx+b

b= y-intercept

Answer:

d on khan

Step-by-step explanation:

Which of the following possible rational roots is a root of the function f(x)=x^3-3x^2+5x-15? *
5 points
-3
-1
1
3

Answers

Given:

The function is

[tex]f(x)=x^3-3x^2+5x-15[/tex]

To find:

The root of the function from the given possible roots.

Solution:

We have,

[tex]f(x)=x^3-3x^2+5x-15[/tex]

At x=-3,

[tex]f(-3)=(-3)^3-3(-3)^2+5(-3)-15[/tex]

[tex]f(-3)=-27-27-15-15[/tex]

[tex]f(-3)=-84\neq 0[/tex]

At x=-1,

[tex]f(-1)=(-1)^3-3(-1)^2+5(-1)-15[/tex]

[tex]f(-1)=-1-3-5-15[/tex]

[tex]f(-1)=-24\neq 0[/tex]

At x=1,

[tex]f(1)=(1)^3-3(1)^2+5(1)-15[/tex]

[tex]f(1)=1-3+5-15[/tex]

[tex]f(1)=-12\neq 0[/tex]

At x=3,

[tex]f(3)=(3)^3-3(3)^2+5(3)-15[/tex]

[tex]f(3)=27-27+15-15[/tex]

[tex]f(3)=0[/tex]

Since, the value of given function is 0 at only x=3, therefore 3 is a root of given function.

Hence, the correct option is D.

Wallace recorded the length of his paintings in feet. Which list shows the lengths in order from greatest to least? List 1: 4.5 feet, 4 feet, 3.99 feet, 3.5 feet, 4.25 feet List 2: 3.99 feet, 4.25 feet, 4.5 feet, 3.5 feet, 4 feet List 3: 4.5 feet, 4.25 feet, 4 feet, 3.99 feet, 3.5 feet List 4: 3.5 feet, 3.99 feet, 4 feet, 4.25 feet, 4.5 feet

Answers

Answer:

list 3

Step-by-step explanation:

if you look at list 3 "4.5 feet, 4.25 feet, 4 feet, 3.99 feet, 3.5 feet" you can easily see that it is going from greatest to smallest because 4.5 is larger than 4.25 and 4.25 is larger than 4 and 4 is larger that 3.99 by 0.01 as you see its only 0.01 diffrence but it is still larger

Answer:

List 3

Step-by-step explanation:

I got it right (;


What type of
solution do you get if
you solve an
equation and the last
line says 12 = 12


A.)No solution
B.)One solution
C.)Infinite solutions

Answers

Answer:

infinite solutions

Step-by-step explanation:

Help please I’m confused

Answers

Answer:

multiple --> the __ of 13 include 13, 26, 39..

factor --> the __ of 10 include 1, 2, 5, 10

in terms of --> solve for variable

domain --> all possible values that can be used as input to the function

range --> all possible values that can be generated by the function (output)

Step-by-step explanation:

this is too much explain im srry just trust me :D

you'll start to understand these eventually   hopefully


Pls helppp this is a grade

Answers

Answer:

9.5

Step-by-step explanation:

9/18 is 1/2

Answer:

19/2 or 9.5

Step-by-step explanation:

452 divided 15 what would be the answer of this? If anyone knows please answer

Answers

Answer:

The answer to this question is given below in the explanation section

Step-by-step explanation:

This is a simple division question.

In division there are some terms used that are:

Dividend: The number which we divide is called a dividend. In this case, 425 is a dividend

Divisor: The number by which we divide is called the divisor. In our question, it is 15.

Quotient: The result obtained is called the quotient.

Remainder: The number left over is called the remainder.

So the answer to this question is:

452/15= 30.133

so the answer is 30.1333

Maricella solves for x in the equation 4 x minus 2 (3 x minus 4) + 4 = negative x + 3 (x + 1) + 1. She begins by adding –4 + 4 on the left side of the equation and 1 + 1 on the right side of the equation. Which best explains why Maricella's strategy is incorrect?

Answers

Answer:

Step-by-step explanation:

Given the equation

4x-2(3x-4)+4 = -x+3(x+1)+1

Open the parenthesis on both sides

4x-6x+8+4 = -x+3x+3+1

-2x+12 = 2x+4

Collect like terms

-2x-2x = 4-12

-4x = -8

Divide both sides by -4

-4x/-4 = -8/-4

x = 2

Hence the value of x is 2

Answer:

4

Step-by-step explanation:

15. 4+3x{2-1]+8 divided [9 divided by7] What is the answer?

Answers

Answer:

86

Step-by-step explanation:

hopes this helps

Given f(x)=−3x−1, solve for x when f(x)=−10.

Answers

Answer: X= -3

Step-by-step explanation: If f(x)= 3x-1

then you would do the math 3x-1= -10

which makes out to be x= -3

NEED HELP
Will give brainiest
What is the y intercept of y= 1/7 x +3

Answers

Answer:

3

y=mx+b

b is the intercept

Answer:

if you look up the last sentence, there is a website with the exact answer

Step-by-step explanation:

BRAINLIEST please? sorry caps lol

what’s the quotient for these numbers??:)

Answers

Answer:

69

Step-by-step explanation:

420

Answer:

1 5/7

Step-by-step explanation:

I need help really fast! Please look at the image below!!
Please and thank you!

Answers

Answer:

119 degrees

Step-by-step explanation:

We know that a full straight line is 180 degrees. This angle is 61 degrees, so we subtract 61 from 180.

180 - 61 = 119 degrees

I hope this helped, please mark Brainliest, thank you!

Answer:

119

Step-by-step explanation:

a line equals 180 degrees, so just subtract 61 from 180

Hurry please help I’m so confused

Answers

Answer: Yes

Step-by-step explanation: You can determine if a graph is a function with the vertical line test. If you drew a vertical line anywhere through this graph it would intersect the graph no more than once.

The graph above is a function.

I need help forming an equation

Answers

Answer:

[tex]\frac{1}{2} d-8=12[/tex]

if f(x)= 3/x^2 - 2 and g(x)=4x then g(f(3))= ?
A.) 3/7
B.) 2/3
C.) 12/3
D.) 12/7
E.) 14/7

Answers

Answer:

https://corbettmaths.files.wordpress.com/2015/03/functions-answers.pdf

Step-by-step explanation:

Sorry if this is not right

perimeter of rectangular plot is 36 meters the length is increased by 6meters and breadth is decreased by 3 meters the area of the plot remains the same what is the length of the plot ​

Answers

Given:

Perimeter of a rectangular plot = 36 m.

Find:

length of the plot

Solution:

Let the length of the plot be x and it's breadth be y.

We know that,

Perimeter of a rectangle = 2(length + breadth)

⟹ 2(x + y) = 36

⟹ x + y = 36/2

⟹ x = 18 - y -- equation (1)

Also,

If length is increased by 6 m & breadth is decreased by 3 m , the area remains same.

Area of a rectangle = length * breadth

So,

(x + 6) * (y - 3) = xy

⟹ x(y - 3) + 6(y - 3) = xy

⟹ xy - 3x + 6y - 18 = xy

Substitute the value of x from equation (1).

⟹ - 3(18 - y) + 6y = 18

⟹ - 54 + 3y + 6y = 18

⟹ 9y = 18 + 54

⟹ y = 72/9

⟹ y = 8 m

Substitute the value of y in equation (1).

⟹ x = 18 - 8

⟹ x = 10 m

Hence,

Length (x) = 10 m

Breadth (y) = 8 m.

∴ The length of the plot is 10 m.

I hope it will help you.

Regards.

Which number is equivalent to 3^4/3^2 (3 to the power of 4 OVER 3 to the power of 2)

A 2

B 9

C 81

D 729

Answers

The answer is Option B

Answer:

B. 9

Can I please be brainliest???

What is the range of these? And what’s the definition of range

Answers

Answer:

-3, 1, 2

Step-by-step explanation:

the range is the y- values

Answer:

The range is the difference between the highest and lowest values in a set of numbers. To find it, subtract the lowest number in the distribution from the highest.

Step-by-step explanation:

Ast week, Shelly rode her bike a total of 30 miles over a three-day period. On the second day, she rode 4 5 the distance she rode on the first day. On the third day, she rode 3 2 the distance she rode on the second day. How many miles did Shelly ride on each day? Select your answers from the drop-down lists. On the first day, Shelly rode miles. On the second day, Shelly rode miles. On the third day, Shelly rode miles.

Answers

Answer: On the first day, Shelly rode 10 miles on first day.

On the second day, Shelly rode 8 miles.

On the third day, Shelly rode 12 miles.

Step-by-step explanation:

Let x = distance rod eon first day.

Shelly rode her bike a total of 30 miles over a three-day period.

Then as per given,

Distance rode on second day = [tex]\dfrac45x[/tex]

Distance rode on third day [tex]=\dfrac32\times\text{ Distance rode on second day}[/tex]

[tex]=\dfrac32\times\dfrac45x=\dfrac{6}{5}x[/tex]

Total distance rode = [tex]x+\dfrac45 x+\dfrac65x=30[/tex]

[tex]\Rightarrow\ \dfrac{5x+4x+6x}{5}=30\\\\\Rightarrow\ \dfrac{15x}{5}=30\\\\\Rightarrow\ 3x=30\\\\\Rightarrow\ x=\dfrac{30}{3}\\\\\Rightarrow\ x=10[/tex]

Hence, She rode 10 miles on first day.

Distance rode on second day = [tex]\dfrac45\times10=4\times2=8\text{ miles}[/tex]

Distance rode on third day [tex]=\dfrac{3}{2}\times8=3\times4=12\text{ miles}[/tex]

A car travels 175 miles using 7 gallons of gas. At that rate, how far can the car travel using 4 gallons of gas?

Answers

Answer:

25

Step-by-step explanation:

175 ÷ 7 =25

Give the equation of the line going through the point (0, -3) and perpendicular to 4x - 8y = 16.

Please answer quickly. It's due tomorrow

Answers

Answer:

y=-1/4x+3

Step-by-step explanation:

put it into y-y1=m(x-x1) form, which gives you y+3=-1/4x(x+0)

(side note: we change the y- to a y+ becuase the y in the point in negative. another side note, the slope is -1/4x because perpendicular slopes are opposite, in this case 4x (or 4/1x) is the opposite of -1/4 x)

then remove the parenthesis. y+3=-1/4x

then isolate y and balance the equation.

y+3-3=-1/4x-3

this gives you the answer of y=-1/4x-3.

Answer:

y=-2x-3

Step-by-step explanation:

The equation of the line in the slope-intercept form is y=x2−2.  

The slope of the perpendicular line is negative inverse: m=−2.

So, the equation of the perpendicular line is y=−2x+a.

To find a, we use the fact that the line should pass through the given point: −3=(−2)⋅(0)+a.

Thus, a=−3.

Therefore, the equation of the line is y=−2x−3.

Answer: y=−2x−3.

Help me please asap!

Answers

Answer:

The student did not distribute correctly

please me proportional or nonproportal ​

Answers

Answer:

looks proportional to me

Step-by-step explanation:

Answer:

Proportional

Step-by-step explanation:

Direct proportion:

y = kx

1*4 = 4

2*4 = 8

3*4 = 12

Given values in the table satisfy the equation of direct proportion. So, it is a direct proportion.

Other Questions
Which process is most responsible for moving heat through the atmosphere? The mean pulse rate (in beats per minute) of adult males is equal to 69 bpm. For a random sample of 162 adult males, the mean pulse rate is 69.6 bpm and the standard deviation is 11.3 bpm. Find the value of the test statistic. Brandon rides his bike 7 miles south and then 24 miles west. About how far is he from his starting point Describe the Triangular Slave Trade including the conditions of the Middle Passage. List all possible values of the magnetic quantum number ml for a 4p election How do you know that K, an alkali metal is very reactive?a. It conducts heatb. It conducts electricityC. It is a soft and shiny metald. It contains one valence electron What should Congress do to government spending to change the demand? Help pleaseee. WAHTS THE ANSWER Solve the system by elimination (please show all the steps!)-2x+2y+3z=0-2x-y+z=-32x+3y+3z=5 What gas is produced in photosynthesis that directly benefits us? LaWanda was born in 2001 and plans to retire at age 64. According to her most recent Social Security statement, her full benefit will be $2,100 per month. What is her monthly benefit if she begins claiming Social Security at age 64?A)$1,430B)$1,640C)$1,560D)$1,680 Convert the following to the nearest tenth. (You may use a calculator.)105 kilometers to miles miles = ? Graph -5Circle is a. openb. closedArrow is pointingc. leftd. right why do you have to pay for unlimited answers thats honestly just dumb :( bc i have no money.. In which field of Information Technology do IT professionals work directly with other employees to solve their general technology related issues? Information Services and Support Network System Administration Programming and Software Development Interactive Media Math the question is 6 6/8 Divided into 2 3/6 + 3 5/6 Brenda is throwing a party. it costs $300 to rent the hall. It costs $35 per person for food. Write an equation to show the total cost of the party (y) based on the number of people invited (x). Simplify. help plz the two is a exponent-3 2 = Please help me the picture is above, just fill in the table Ill mark you as brainliest if you answer. Thank you :) Triangle ABC is translated 2 units up and 4 units to the right and is then rotated 180 toproduce the image triangle A'B'C'. What are the coordinates of B' in the resulting figure?